Đến nội dung

Hình ảnh

Topic tổng hợp một số bất đẳng thức trong kì thi MO các nước

* * * * * 16 Bình chọn

  • Please log in to reply
Chủ đề này có 501 trả lời

#201
khanghaxuan

khanghaxuan

    Trung úy

  • Thành viên
  • 969 Bài viết

Bài 79 : ( Turkey JBMO 2014 ) Cho $a,b,c>0$ thỏa mãn : $a+b+c+abc=4$ . CMR : $(1+\frac{a}{b}+ca)(1+\frac{b}{c}+ab)(1+\frac{c}{a}+bc)\geq 27$

Spoiler

Bài 80 : Cho $x,y,z$ là các số thực thỏa mãn : $x+y+z=0$ và $x^{2}+y^{2}+z^{2}=6$. Tìm GTLN của : 

$\begin{vmatrix} (x-y)(y-z)(z-x) \end{vmatrix}$

Bài 81 : (JBMO 2013) Cho $a,b>0$ thỏa mãn : $ab\geq 1$ . CMR : $(a+2b+\frac{2}{a+1})(b+2a+\frac{2}{b+1})\geq 16$

Bài 82 : Cho $a,b,c>0$ . CMR : $\sum \frac{a^{5}}{b(a^{4}+b^{4}+a^{3}c)}\geq 1$

Spoiler


Bài viết đã được chỉnh sửa nội dung bởi khanghaxuan: 06-06-2015 - 15:59

Điều tôi muốn biết trước tiên không phải là bạn đã thất bại ra sao mà là bạn đã chấp nhận nó như thế nào .

- A.Lincoln -

#202
hoanglong2k

hoanglong2k

    Trung úy

  • Điều hành viên THCS
  • 965 Bài viết

Bài 79 : ( Turkey JBMO 2014 ) Cho $a,b,c>0$ thỏa mãn : $a+b+c+abc=4$ . CMR : $(1+\frac{a}{b}+ca)(1+\frac{b}{c}+ab)(1+\frac{c}{a}+bc)\geq 27$

Spoiler

 

 Áp dụng AM-GM : $4=a+b+c+abc\geq 4\sqrt[4]{(abc)^2}\Rightarrow abc\leq 1$

 Ta có : BĐT $\Leftrightarrow \prod (abc+a+b)\geq 27abc$

                     $\Leftrightarrow \prod (4-a)\geq 27abc$

                     $\Leftrightarrow 64-16\sum a+4\sum ab-28abc\geq 0$

                     $\Leftrightarrow 4\sum ab-12abc\geq 0$

                     $\Leftrightarrow \sum ab\geq 3abc$

 Áp dung AM-GM lại có : $ab+bc+ca\geq 3\sqrt{(abc)^2}\geq 3abc$

 Vậy có điều cần chứng minh

 Dấu "=" xảy ra khi $a=b=c=1$



#203
datmc07061999

datmc07061999

    Trung sĩ

  • Thành viên
  • 198 Bài viết

 

Bài 80 : Cho $x,y,z$ là các số thực thỏa mãn : $x+y+z=0$ và $x^{2}+y^{2}+z^{2}=6$. Tìm GTLN của : 

$\begin{vmatrix} (x-y)(y-z)(z-x) \end{vmatrix}$

 

  Cách của em hơi trâu bò.... :(

    Mong mọi người chỉ giáo... :icon6:

  Từ giả thiết, ta có $xy+yz+zx=-3$

    $P^{2}=(x-y)^{2}(y-z)^{2}(z-x)^{2}= (12-3z^{2})(12-3y^{2})(12-3x^{2})$. ( Do $xy+yz+zx=-3\Rightarrow xy=-3-z(x+y)=z^{2}-3$ và $x^{2}+y^{2}=6-z^{2}$)

  Khai triển và sử dụng giả thiết ta được: $P^{2}=108(x^{2}y^{2}+y^{2}z^{2}+z^{2}x^{2})-27(xyz)^{2}-864\leq 108$

                                 (Do $(xy+yz+zx)^{2}=9\Leftrightarrow x^{2}y^{2}+y^{2}z^{2}+z^{2}x^{2}+2xyz(x+y+z)=9\Rightarrow x^{2}y^{2}+y^{2}z^{2}+z^{2}x^{2}=9$)

  Vậy $\left | P \right |\leq \sqrt{108}\Rightarrow P\leq 6\sqrt{3}$

       Dấu $"="$ xảy ra $\Leftrightarrow \left\{\begin{matrix} x+y+z=0 & \\ x^{2}+y^{2}+z^{2}=6 & \\ xyz=0 & \end{matrix}\right.\Leftrightarrow (x,y,z)=(\sqrt{3};-\sqrt{3};0)$

                và các hoán vị...

---------------------------------

@ducvipdh12: không sao,cách nào cũng được ghi nhận cả


Bài viết đã được chỉnh sửa nội dung bởi ducvipdh12: 05-06-2015 - 22:12

Hãy cố gắng vượt qua tất cả dù biết mình chưa là gì...


#204
datmc07061999

datmc07061999

    Trung sĩ

  • Thành viên
  • 198 Bài viết

Bài 81 : (JBMO 2013) Cho $a,b>0$ thỏa mãn : $ab\geq 1$ . CMR : $(a+2b+\frac{2}{a+1})(b+2a+\frac{2}{b+1})\geq 16$

  Quy đồng ta được:

      $(a^{2}+a+2ab+2b+2)(b^{2}+b+2ab+2a+2)\geq 16(ab+a+b+1)$

      $\Leftrightarrow (3a+2b+3)(3b+2a+3)\geq 16(ab+a+b+1)$        (Để ý $a^{2}+1\geq 2a, ab\geq 1$)

      $\Leftrightarrow 6a^{2}+6b^{2}\geq 3ab+a+b+7$

  Mặt khác:  $\frac{3a^{2}}{2}+\frac{3b^{2}}{2}\geq 3ab$

                   $\frac{7a^{2}}{2}+\frac{7b^{2}}{2}\geq 7ab\geq 7$

                   $a^{2}+b^{2}\geq 2a+2b-2\geq a+b+2\sqrt{ab}-2\geq a+b$

  Vậy ta có ĐPCM.

  

  :excl: Các bạn like ủng hộ mình nha...


Bài viết đã được chỉnh sửa nội dung bởi datmc07061999: 06-06-2015 - 07:55

Hãy cố gắng vượt qua tất cả dù biết mình chưa là gì...


#205
khanghaxuan

khanghaxuan

    Trung úy

  • Thành viên
  • 969 Bài viết

Lời giải bài 80 :  

Không mất tính tổng quát ta giả sử : $x\geq y\geq z$ 

Do đó : $P=(x-y)(y-z)(x-z)$

Mặt khác ta có : $(x-y)(y-z)\leq (\frac{x-z}{2})^{2}\Rightarrow (x-y)(y-z)(x-z)\leq \frac{(x-z)^{3}}{4}$

Tiếp theo ta sẽ tìm GTLN của : $x-z$ . Thật vậy , ta có: 

$3y^{2}\geq 0\Leftrightarrow 2y^{2}+(x+z)^{2}\geq 0\Leftrightarrow 2(x^{2}+y^{2}+z^{2})\geq x^{2}+z^{2}-2xz\Leftrightarrow (x-z)^{2}\leq 12\Rightarrow x-z\leq 2\sqrt{3}$

Do dó : $P\leq \frac{(2\sqrt{3})^{3}}{4}=6\sqrt{3}$

Dấu $=$ xảy ra khi : $\left\{\begin{matrix} x=\sqrt{3} & & \\ y=0 & & \\ z=-\sqrt{3} & & \end{matrix}\right.$


Điều tôi muốn biết trước tiên không phải là bạn đã thất bại ra sao mà là bạn đã chấp nhận nó như thế nào .

- A.Lincoln -

#206
khanghaxuan

khanghaxuan

    Trung úy

  • Thành viên
  • 969 Bài viết

Lời giải bài 81 : 

Ta có : $\frac{a+1}{2}+\frac{2}{a+1}\geq 2\Rightarrow \frac{a}{2}+\frac{2}{a+1}\geq \frac{3}{2}\Rightarrow a+2b+\frac{2}{a+1}\geq \frac{a}{2}+\frac{3}{2}+2b$

Tương tự ta cũng có : $b+2a+\frac{2}{b+1}\geq \frac{b}{2}+2a+\frac{3}{2}$

Do đó ta cần chứng minh : $(2b+\frac{a}{2}+\frac{3}{2})(2a+\frac{b}{2}+\frac{3}{2})\geq 16$

Thật vậy , áp dụng Cauchy - Schawrz ta được : $(2b+\frac{a}{2}+\frac{3}{2})(2a+\frac{b}{2}+\frac{3}{2})\geq (2\sqrt{ab}+\frac{\sqrt{ab}}{2}+\frac{3}{2})^{2}\geq 4^{2}=16$ (ĐPCM :)) )

 

P/s : Bài này có rất nhiều cách giải :)


Điều tôi muốn biết trước tiên không phải là bạn đã thất bại ra sao mà là bạn đã chấp nhận nó như thế nào .

- A.Lincoln -

#207
khanghaxuan

khanghaxuan

    Trung úy

  • Thành viên
  • 969 Bài viết

Lời giải bài 82 : Cho $a,b,c>0$ . CMR : $\sum \frac{a^{5}}{b(a^{4}+b^{4}+a^{3}c)}\geq 1$

Với các kiểu bất đẳng thức dạng phân thức này thì theo kinh nghiệm của mình ta nên áp dụng Cauchy- Schawrz dạng Engel . Để ý thấy tử số của mỗi phân thức bậc lẻ nên để áp dụng Cauchy- Schawrz dạng Engel thì ta sẽ làm cho bậc từ lẻ $\rightarrow$ chẵn . 

Cụ thể ta sẽ biến đổi như sau : $\sum \frac{a^{5}}{b(a^{4}+c^{4}+a^{3}c)}=\sum \frac{a^{6}}{ab(a^{4}+c^{4}+a^{3}c)}$

Sau khi đã làm cho tử số như ta muốn thì tiếp theo ta sẽ áp dụng Cauchy- Schawrz dạng Engel như sau : 

$\sum \frac{a^{6}}{ab(a^{4}+c^{4}+a^{3}c)}\geq \frac{(\sum a^{3})^{2}}{\sum ab(a^{4}+c^{4}+a^{3}c)}=\frac{(\sum a^{3})^{2}}{\sum a^{5}b+\sum ab^{5}+abc\sum a^{3}}$

Tới đây ta sẽ tìm mọi cách để đánh giá mẫu số ( Vì nhìn tử số thấy chẳng có gì để khai thác cả :P )

Chú ý rằng : $\sum a^{5}b+\sum ab^{5}+abc\sum a^{3}=\sum ab(a^{4}+b^{4})+abc\sum a^{3}=\sum (ab(a^{4}+b^{4})+abc.c^{3})=\sum ab(a^{4}+b^{4}+c^{4})=(\sum a^{4})(\sum ab)$

Do đó ta quy về chứng minh : $\frac{(\sum a^{3})^{2}}{(\sum ab)(\sum a^{4})}\geq 1$

Hay ta cần chứng minh: $(a^{3}+b^{3}+c^{3})^{2}\geq (ab+bc+ca)(a^{4}+b^{4}+c^{4})(*)$ 

Lướt nhìn qua thì ta có thể khẳng định đây là một bđt hay ,đẹp ,khá mạnh và được dùng làm bổ đề để có thể sáng tạo ra các bđt khác mang tính thách thức cao hơn . 

Phần chứng minh $(*)$ đã có ở trong box BĐT Olympic nhưng minh xin nêu lại để thấy rõ rằng : chứng minh nó quả không dễ dàng

Spoiler

$(\sum a^{3})^{2}\geq (\sum a^{4})(\sum ab)$

$\Leftrightarrow \sum a^{6}+2\sum a^{3}b^{3}\geq \sum ab(a^{4}+b^{4})+abc\sum a^{3}$

$\Leftrightarrow (\sum a^{6}-3(abc)^{2})+2(\sum a^{3}b^{3}-3(abc)^{2})\geq abc(\sum a^{3}-3abc)+\sum ab(a^{4}+b^{4})-6(abc)^{2}$

$\Leftrightarrow \frac{(\sum a^{2})}{2}(\sum (a^{2}-b^{2})^{2})+(\sum ab)(\sum (bc-ca)^{2})\geq \frac{abc(\sum a)}{2}(\sum (a-b)^{2})+(\sum ab(a^{4}+b^{4})-2\sum a^{3}b^{3})+2\sum (ab)^{3}-6(abc)^{2}$

$\Leftrightarrow \Leftrightarrow \frac{(\sum a^{2})}{2}(\sum (a^{2}-b^{2})^{2})+(\sum ab)(\sum (bc-ca)^{2})\geq \frac{abc(\sum a)}{2}(\sum (a-b)^{2})+\sum ab(a^{2}-b^{2})^{2}+(\sum ab)(\sum (bc-ca)^{2})$

$\Leftrightarrow \sum (a-b)^{2}(\frac{(\sum a^{2})(a+b)^{2}}{2}+c^{2}\sum ab-\frac{abc(\sum a)}{2}-ab(a+b)^{2}-c^{2}(\sum ab))\geq 0$

$\Leftrightarrow \sum (a-b)^{2}(\frac{(a+b)^{2}((a-b)^{2}+c^{2})}{2})-\frac{abc(\sum a)}{2}\geq 0$

$\Leftrightarrow \sum (a-b)^{2}((a^{2}-b^{2})^{2}+c^{2}(a+b)^{2}-abc\sum a)\geq 0$

Mà bđt cuối này thì luôn đúng :)) 


Điều tôi muốn biết trước tiên không phải là bạn đã thất bại ra sao mà là bạn đã chấp nhận nó như thế nào .

- A.Lincoln -

#208
binhnhaukhong

binhnhaukhong

    Sĩ quan

  • Thành viên
  • 343 Bài viết

 

 

Spoiler

 

Khó có lời giải khác đẹp em ạ S.O.S đã là tối ưu rồi dúng cái bổ đề về đại lượng $(a-b)(b-c)(c-a)$ thì cũng ra nhưng tính toán thì hơi mệt $BW$ thì càng không nên.  :unsure:


Bài viết đã được chỉnh sửa nội dung bởi ducvipdh12: 06-06-2015 - 20:24

Quy Ẩn Giang Hồ. 

So goodbye!

 

:off:  :off:  :off:  :off:  :off:  :off: 


#209
Belphegor Varia

Belphegor Varia

    Thượng sĩ

  • Thành viên
  • 227 Bài viết

Bài 83 (VMO 1999) : Cho các số thực dương $a,b,c$ thỏa mãn điều kiện $abc+a+c=b$ . Tìm GTLN của :
$P=\frac{2}{a^{2}+1}-\frac{2}{b^{2}+1}+\frac{3}{c^{2}+1}$
Bài 84 (Thụy Điển 1985) : Cho $a>b>0$ . Chứng minh bất đẳng thức sau:
$\frac{(a-b)^{2}}{8a}<\frac{a+b}{2}-\sqrt{ab}<\frac{(a-b)^{^{2}}}{8b}$


Bài viết đã được chỉnh sửa nội dung bởi khanghaxuan: 06-06-2015 - 18:37

$ \textbf{NMQ}$

Wait a minute, You have enough time. Also tomorrow will come 

Just take off her or give me a ride 

Give me one day or one hour or just one minute for a short word 

 


#210
hoctrocuaHolmes

hoctrocuaHolmes

    Thượng úy

  • Thành viên
  • 1013 Bài viết

Bài 84 (Thụy Điển 1985) : Cho $a>b>0$ . Chứng minh bất đẳng thức sau:
$\frac{(a-b)^{2}}{8a}<\frac{a+b}{2}-\sqrt{ab}<\frac{(a-b)^{^{2}}}{8b}$

Chứng minh vế thứ nhất:$\frac{(a-b)^{2}}{8a}<\frac{a+b}{2}-\sqrt{ab}$

Ta có $\frac{(a-b)^{2}}{8a}<\frac{a+b}{2}-\sqrt{ab}\Leftrightarrow (\frac{a-b}{\sqrt{8a}})^{2}< \frac{a+b-2\sqrt{ab}}{2}=\frac{(\sqrt{a}-\sqrt{b})^{2}}{2}=(\frac{\sqrt{a}-\sqrt{b}}{\sqrt{2}})$ (vì $a>b>0$)

$\Leftrightarrow \frac{a-b}{\sqrt{8a}}< \frac{\sqrt{a}-\sqrt{b}}{\sqrt{2}}\Leftrightarrow \sqrt{2}(a-b)< (\sqrt{a}-\sqrt{b})\sqrt{8a}\Leftrightarrow \sqrt{2}a-\sqrt{2}b< \sqrt{8}a-\sqrt{8ab}$

$\Leftrightarrow a-b< 2a-2\sqrt{ab}\Leftrightarrow a-2\sqrt{ab}+b> 0\Leftrightarrow (\sqrt{a}-\sqrt{b})^{2}> 0$ (luôn đúng)

Vậy,bất đẳng thức đã được chứng minh

Chứng minh vế thứ hai: $\frac{a+b}{2}-\sqrt{ab}<\frac{(a-b)^{^{2}}}{8b}$

Hoàn toàn tương tự ta biến đổi thành $\frac{\sqrt{a}-\sqrt{b}}{\sqrt{2}}< \frac{a-b}{\sqrt{8b}}\Leftrightarrow \sqrt{a}-\sqrt{b}< \frac{a-b}{2\sqrt{b}}\Leftrightarrow 2\sqrt{ab}-2b< a-b\Leftrightarrow a+b-2\sqrt{ab}> 0\Leftrightarrow (\sqrt{a}-\sqrt{b})^{2}> 0$

(luôn đúng)

Vậy bất đẳng thức đã được chứng minh


Bài viết đã được chỉnh sửa nội dung bởi votruc: 06-06-2015 - 17:15


#211
khanghaxuan

khanghaxuan

    Trung úy

  • Thành viên
  • 969 Bài viết

Lời giải bài 83 : Cho các số thực dương $a,b,c$ thỏa mãn điều kiện $abc+a+c=b$ . Tìm GTLN của :

$P=\frac{2}{a^{2}+1}-\frac{2}{b^{2}+1}+\frac{3}{c^{2}+1}$

Thoạt đầu , ta nhìn $P$ không phải là một đa thức đối xứng hay hoán vị . Do đó gần nên chắc chắn rằng dấu $=$ sẽ xảy ra lệch nhau . Điều đó càng được khẳng định rõ nét hơn qua điều kiện . Vì vậy , để bắt đầu giải bài toán này chúng ta phải khai thác tối đa giả thiết mà đề cho .

$abc+a+c=b\Rightarrow b=\frac{a+c}{1-ac}$

Tới đây thì chúng ta có thể thế vào rồi tính theo 2 biến $a$ và $c$ , tuy nhiên nếu tinh ý hơn ta sẽ để ý đến công thức lượng giác sau :    $tan(A+C)=\frac{tanA+tanC}{1-tanA.tanC}$

Do đó , một cách tương ứng ta đặt : $\left\{\begin{matrix} a=tanA & \\ c=tanC & \end{matrix}\right.\Rightarrow b=tan(A+C)$

Nên ta cần tìm GTLN của : $2cos^{2}A+3cos^{2}C-2cos^{2}(A+C)$

Tiếp theo , ta để ý rằng : $cos^{2}A-cos^{2}(A+C)=sinC.sin(2A+C)$

Do đó ta có : $P=2sin C.sin (2A+C)+3(1-sin^{2}C)\leq 2sin C-32sin^{2}C+3\leq \frac{10}{3}$

Vì vậy , $P_{max}=\frac{10}{3}\Leftrightarrow \left\{\begin{matrix} a=\frac{\sqrt{2}}{2} & & \\ b=\sqrt{2} & & \\ c=\frac{\sqrt{2}}{4} & & \end{matrix}\right.$


Điều tôi muốn biết trước tiên không phải là bạn đã thất bại ra sao mà là bạn đã chấp nhận nó như thế nào .

- A.Lincoln -

#212
ducvipdh12

ducvipdh12

    Sĩ quan

  • Thành viên
  • 454 Bài viết

Bài 85: (Crux Mathematical) Cho $ a, b $ là các số thực thỏa $ \sqrt[3]{a}+\sqrt[3]{b}=\sqrt[3]{b-\dfrac{1}{4}} $. Chứng minh rằng $ -1 \leq a < 0 $.

hình như bạn chưa đọc kỹ tiêu đề của topic,đây là topic về các bài BĐT trong các kì thi MO các nước chơ không phải xuất phải từ 1 tạp chí như Crux,THTT,...

@Khanghaxuan : Theo em thì những bài trong các tạp chí trên mà có tính ứng dụng thì cũng nên post . Chứ em thấy mấy bài  MO hầu hết cũng chỉ là hệ quả của mấy bài toán gốc .Còn về bài toán trên thì nên ban :P 


Bài viết đã được chỉnh sửa nội dung bởi khanghaxuan: 07-06-2015 - 07:33

FAN THẦY THÔNG,ANH CẨN,THẦY VINH :icon6: :icon6:

#213
Belphegor Varia

Belphegor Varia

    Thượng sĩ

  • Thành viên
  • 227 Bài viết

Bài 86 (Vô Địch các nước Bắc Phi-86) : Cho $a,b,c>0$ . Chứng minh bất đẳng thức :
$3(\frac{1}{ab}+\frac{1}{bc}+\frac{1}{ca})\geq 4(\frac{1}{a+b}+\frac{1}{b+c}+\frac{1}{c+a})^{2}$
Bài 87 (VMO 2002) : Cho $a,b,c$ là 3 số thực tùy ý. Chứng minh rằng :
$6(a+b+c)(a^{2}+b^{2}+c^{2})\leq 27abc+10(a^{2}+b^{2}+c^{2})^{\frac{3}{2}}$
Hỏi dấu đẳng thức xảy ra khi nào?


$ \textbf{NMQ}$

Wait a minute, You have enough time. Also tomorrow will come 

Just take off her or give me a ride 

Give me one day or one hour or just one minute for a short word 

 


#214
hoanglong2k

hoanglong2k

    Trung úy

  • Điều hành viên THCS
  • 965 Bài viết

Bài 86 (Vô Địch các nước Bắc Phi-86) : Cho $a,b,c>0$ . Chứng minh bất đẳng thức :
$3(\frac{1}{ab}+\frac{1}{bc}+\frac{1}{ca})\geq 4(\frac{1}{a+b}+\frac{1}{b+c}+\frac{1}{c+a})^{2}$

 Bài 86

 Ta có : $3\sum \frac{1}{ab}\geq 12\sum \frac{1}{(a+b)^2}\geq 4\left ( \sum \frac{1}{a+b} \right )^2$

 

Spoiler



#215
hoanglong2k

hoanglong2k

    Trung úy

  • Điều hành viên THCS
  • 965 Bài viết

Bài 87 (VMO 2002) : Cho $a,b,c$ là 3 số thực tùy ý. Chứng minh rằng :
$6(a+b+c)(a^{2}+b^{2}+c^{2})\leq 27abc+10(a^{2}+b^{2}+c^{2})^{\frac{3}{2}}$
Hỏi dấu đẳng thức xảy ra khi nào?

 Bất đẳng thức đồng bậc 3 nên ta chuẩn hoá $a^2+b^2+c^2=9$

 Khi đó : BĐT $\Leftrightarrow 2(a+b+c)-abc\leq 10\Leftrightarrow a(2-bc)+(b+c).2\leq 10$

 Giả sử $a\geq b\geq c \Rightarrow |bc|\leq \frac{9-a^2}{2}\leq 3 \Rightarrow bc\in [-3;3]$

 Áp dụng Cauchy-Schwarz ta có : $a(2-bc)+(b+c).2\leq \sqrt{(9+2bc)(b^2c^2-4bc+8)}\leq 10\Leftrightarrow (bc+2)^2(2bc-7)\leq 0$

 Luôn đúng $\forall ~bc\in [-3;3]$

 Từ đó có đpcm



#216
loigiailanhlung

loigiailanhlung

    Hạ sĩ

  • Thành viên
  • 56 Bài viết

Bài 88:
Cho $x_{1},x_{2},...,x_{n}>0$ thỏa mãn điều kiện $x_{k+1}\geq x_{1}+x_{2}+...x_{n} \forall k$
Hãy tìm min của hằng số c sao cho:
$$\sqrt{x_{1}}+\sqrt{x_{2}}+...\sqrt{x_{n}} \leq c.\sqrt{x_{1}+x_{2}+...+x_{n}}$$


Bài viết đã được chỉnh sửa nội dung bởi khanghaxuan: 07-06-2015 - 11:04


#217
longatk08

longatk08

    Sĩ quan

  • Thành viên
  • 350 Bài viết

Bài 89: (Turkish MO 2007)

Cho $a,b,c$ thực dương thỏa mãn: $a+b+c=3$.Chứng minh bất đẳng thức:

 

$\frac{a^2+3b^2}{ab^2(4-ab)}+\frac{b^2+3c^2}{bc^2(4-bc)}+\frac{c^2+3a^2}{ca^2(4-ca)}\geq 4$



#218
Hoang Tung 126

Hoang Tung 126

    Thiếu tá

  • Thành viên
  • 2061 Bài viết

Bài 89: (Turkish MO 2007)

Cho $a,b,c$ thực dương thỏa mãn: $a+b+c=3$.Chứng minh bất đẳng thức:

 

$\frac{a^2+3b^2}{ab^2(4-ab)}+\frac{b^2+3c^2}{bc^2(4-bc)}+\frac{c^2+3a^2}{ca^2(4-ca)}\geq 4$

Theo Cosi ta có :

 

 $\sum \frac{a^2+3b^2}{ab^2(4-ab)}=\sum \frac{(a^2+b^2)+2b^2}{ab^2(4-ab)}\geq \sum \frac{2ab+2b^2}{ab^2(4-ab)}=\sum \frac{2b(a+b)}{ab^2(4-ab)}$

$=2\sum \frac{a+b}{ab(4-ab)}\geq 2\sum \frac{2\sqrt{ab}}{ab(4-ab)}=4\sum \frac{1}{\sqrt{ab}(4-ab)}$   (1)

 

Mà $8+ab=(4-ab)+(4-ab)+3ab\geq 3\sqrt[3]{3ab(4-ab)^2}= > (8+ab)^3\geq 81ab(4-ab)^2$

$= > ab(4-ab)^2\leq \frac{(8+ab)^3}{81}= > \sqrt{ab}(4-ab)\leq \frac{\sqrt{(8+ab)^3}}9{}= > \frac{1}{\sqrt{ab}(4-ab)}\geq \frac{9}{\sqrt{(8+ab)^3}}$

$= > \sum \frac{1}{\sqrt{ab}(4-ab)}\geq 9\sum \frac{1}{\sqrt{(8+ab)^3}}\geq 9.3\sqrt[3]{\frac{1}{\sqrt{(8+ab)^3(8+bc)^3(8+ac)^3}}}=\frac{27}{\sqrt{(8+ab)(8+bc)(8+ac)}}\geq \frac{27}{\sqrt{\frac{(8+ab+8+bc+8+ac)^3}{27}}}\geq \frac{27}{\sqrt{\frac{(24+\frac{(a+b+c)^2}{3})}{27}}}=\frac{27}{\sqrt{\frac{(25+3)^3}{27}}}=\frac{27}{\sqrt{27^2}}=1$

   $= > \sum \frac{1}{\sqrt{ab}(4-ab)}\geq 1$    (2)

 

   Từ (1) ,(2) $= > \sum \frac{a^2+3b^2}{ab^2(4-ab)}\geq 4$ và ta có ĐPCM

 

Dấu = xảy ra khi $a=b=c=1$


Bài viết đã được chỉnh sửa nội dung bởi Hoang Tung 126: 08-06-2015 - 19:47


#219
dogsteven

dogsteven

    Đại úy

  • Thành viên
  • 1567 Bài viết

 Bất đẳng thức đồng bậc 3 nên ta chuẩn hoá $a^2+b^2+c^2=9$

 Khi đó : BĐT $\Leftrightarrow 2(a+b+c)-abc\leq 10\Leftrightarrow a(2-bc)+(b+c).2\leq 10$

 Giả sử $a\geq b\geq c \Rightarrow |bc|\leq \frac{9-a^2}{2}\leq 3 \Rightarrow bc\in [-3;3]$

 Áp dụng Cauchy-Schwarz ta có : $a(2-bc)+(b+c).2\leq \sqrt{(9+2bc)(b^2c^2-4bc+8)}\leq 10\Leftrightarrow (bc+2)^2(2bc-7)\leq 0$

 Luôn đúng $\forall ~bc\in [-3;3]$

 Từ đó có đpcm

Ta có một lời giải rất xấu bằng phân tách trường hợp như sau:

Chuẩn hóa $a^2+b^2+c^2=9$. Giả sử $a\geqslant b\geqslant c$

Nếu $a\leqslant 0$ thì bất đẳng thức hiển nhiên đúng do $abc>-10$

Nếu $a\geqslant 0$ và $b,c\leqslant 0$ thì $2(a+b+c)\leqslant 2a\leqslant 6\leqslant 10\leqslant 10+abc$

Nếu $a\geqslant b\geqslant 0\geqslant c$ thì $2a+2b+2c\le 9+3c$ nên ta cần chứng minh $3c\le 1+\dfrac{c(9-c^2)}{2}$ luôn đúng.

Nếu $c\geqslant 0$ thì $2a+2b+2c\leqslant 9+c$

- Trường hợp $c\leqslant 1$ thì $9+c\leqslant 10\leqslant 10+abc$

- Trường hợp $c\geqslant 1$ thì $9+c\leqslant 9+abc\leqslant 10+abc$


Bài viết đã được chỉnh sửa nội dung bởi dogsteven: 08-06-2015 - 20:08

Quyết tâm off dài dài cày hình, số, tổ, rời rạc.


#220
dogsteven

dogsteven

    Đại úy

  • Thành viên
  • 1567 Bài viết

Bài 59:(PP điểm rơi trong AM-GM)(VNTST 2001)

Cho các số thực dương $a,b,c$ thỏa mãn: $2x+4y+7z=2xyz$. Tìm giá trị nhỏ nhất của biểu thức: $P=x+y+z$

 

p/s: trình bày luôn cách tìm điểm rơi nhé!

Ta dùng nhân tử để tìm điểm rơi.

Xét hàm số $L=x+y+z+\lambda(2x+4y+7z-2xyz)$

Đạo hàm riêng cho biến $x$ được $1=2\lambda (yz-1)$

Đạo hàm riêng cho biến $y$ được $1=2\lambda(zx-2)$

Đạo hàm riêng cho biến $z$ được $1=\lambda(2xy-7)$

Do đó $2(yz-1)=2(zx-2)=2xy-7$


Quyết tâm off dài dài cày hình, số, tổ, rời rạc.





1 người đang xem chủ đề

0 thành viên, 1 khách, 0 thành viên ẩn danh